Add solutions for Serway and Jewett v8's chapter 31.
[course.git] / latex / problems / Serway_and_Jewett_8 / problem31.30.tex
1 \begin{problem*}{31.30}
2 A rectangular coil with resistance $R$ has $N$ turns, each of length
3 $l$ and width $w$ as shown in Figure~P31.30.  The coil moves into a
4 uniform magnetic field $\vect{B}$ with constant velocity $\vect{v}$.
5 What are the magnitude and direction of the total magnetic force on
6 the coil \Part{a} as it enters the magnetic field, \Part{b} as it
7 moves within the field, and \Part{c} as it leaves the field?
8 \begin{center}
9 %          x x x Bin
10 %  +--+ v  x x x x
11 % w|  |->  x x x x
12 %  +--+    x x x x
13 %   l      x x x x
14 \begin{asy}
15 import Mechanics;
16 import ElectroMag;
17
18 real w = 5mm;
19 real l = 6mm;
20 real u = 1.5cm;
21
22 Vector B = BField(phi=-90);
23 vector_field((l/2 + 7mm + u/2, 0), width=u, height=u, v=B, outline=dashed);
24 Vector v = Velocity((l/2, 0), "$\vect{v}$");  v.draw();
25 path p = scale(l, w)*shift((-0.5, -0.5))*unitsquare;
26 draw(p);
27 draw(shift(1.5pt*dir(-45))*p);
28 Distance Dl = Distance((-l/2, -w/2), (l/2, -w/2), "$l$", offset=6pt);
29 Dl.draw();
30 Distance Dw = Distance((-l/2, w/2), (-l/2, -w/2), Label("$w$", embed=Shift),
31     offset=6pt);
32 Dw.draw();
33 \end{asy}
34 \end{center}
35 \end{problem*}
36
37 \begin{solution}
38 \Part{a}
39 If we define ``into the page'' as the positive direction, the flux
40 through the loop will be increasing as the coil enters the field,
41 which will induce a current in the counter-clockwise direction
42 opposing the changing flux.  The right side of the coil and portions
43 of the top and bottom sides will be in the field regions, and because
44 of the current will be subject to magnetic forces directed to the
45 left, down, and up respectively.  Because equal portions of the top
46 and bottom side will be in the field, there will be no vertical
47 component in the net force, which will be directed to the left.
48
49 The magnitude of the induced \EMF\ is
50 \begin{equation}
51   |\EMF| = |-\deriv{t}{\Phi_B}| = \deriv{t}{AB} = B\deriv{t}{Nxw}
52     = NBw\deriv{t}{x} = NBwv \;.
53 \end{equation}
54 This leads to a current of
55 \begin{equation}
56   I = \frac{\EMF}{R} = \frac{Bwv}{R} \;,
57 \end{equation}
58 which causes a magnetic force of
59 \begin{equation}
60   F_B = NIwB\sin(90\dg) = \ans{\frac{N^2B^2w^2v}{R}} \;.
61 \end{equation}
62
63 \Part{b}
64 While the coil is completely inside the field, the flux remains
65 constant, so there is no induced current and \ans{no magnetic force}.
66
67 \Part{c}
68 As the coil leaves the field, the flux drops back towards zero,
69 indicing a clockwise current trying to keep the flux up.  Again, the
70 vertical components of the resulting magnetic force cancel out, but
71 the upward current in the left side will be subject to a magnetic
72 force directed to the left.  The magnitude is the same as
73 for \Part{a}.
74 \end{solution}